Mathcenter Forum

Mathcenter Forum (https://www.mathcenter.net/forum/index.php)
-   ปัญหาคณิตศาสตร์ทั่วไป (https://www.mathcenter.net/forum/forumdisplay.php?f=1)
-   -   โจทย์ปัญหาคณิตศาสตร์ สวัสดีปีใหม่ 2548 ครับ (https://www.mathcenter.net/forum/showthread.php?t=724)

nooonuii 31 ธันวาคม 2004 22:39

โจทย์ปัญหาคณิตศาสตร์ สวัสดีปีใหม่ 2548 ครับ
 
ขอสวัสดีปีใหม่กับสมาชิกทุกท่านด้วยปัญหาคณิตศาสตร์ต่อไปนี้ครับ :)

1. ให้ $a$ เป็นจำนวนตรรกยะในช่วงเปิด $(0,1)$ จงแสดงว่า รากที่ $2548$ ของ $1 - a^{2548}$ เป็นจำนวนอตรรกยะ

2. จงหาจำนวนคำตอบทั้งหมดของปัญหาหนอนแทะต่อไปนี้

FOUR + FIVE = NINE

เมื่ออักษรแต่ละตัวแทนเลขโดดที่แตกต่างกัน

3. จงหาจำนวนจริงทั้งหมดซึ่งสอดคล้องอสมการ $[x]^2\leq x$ เมื่อ $[x]$ แทนจำนวนเต็มที่มากที่สุดซึ่งมีค่าน้อยกว่าหรือเท่ากับ $x$

4. จงหาจำนวนเต็ม $x,y$ ทั้งหมดที่สอดคล้องสมการ $3^x+ 1 = 13y$

5. ให้ $S$ เป็นเซตของจำนวนจริงซึ่งมีสมาชิกเป็นจำนวนจำกัด โดยมีคุณสมบัติว่า ถ้า $a,b\in S$ แล้ว $ab\in S$ ด้วย จงหาเซต $S$ ที่เป็นไปได้ทั้งหมด

6. จงพิสูจน์ว่า $(1 + 2 + ... + 2547)(1 + 1/2 + ... + 1/2547) > 2548^2$

<aaaa> 31 ธันวาคม 2004 23:18

เฉลย ข้อ 1
ใช้ Fermat Last Theorem สมมติว่า 1-a^2548=b^2548 (b เป็นจำนวนตรรกยะ)
หรือ 1=a^2548+b^2548
โดยการคูณด้วยส่วนของ a^2548 และ b^2548 จะได้สมการ c^2548=x^2548+y^2548
(x,y,c เป็นจำนวนเต็ม)
ดังนั้นโดย FLT x=y=c=0 นั่นคือ a=0 ขัดแย้งกับ 0<a<1

TOP 31 ธันวาคม 2004 23:23

สวัสดีปีใหม่ ๒๕๔๘ หวังว่าปีนี้ทุกคนจะได้พัฒนาตนเองในทุกๆด้านนะครับ :)

<aaaa> 31 ธันวาคม 2004 23:29

เฉลยข้อ 4
คิด mod 13 จะเห็นว่าไม่มี solution

gon 31 ธันวาคม 2004 23:33

ว่าจะหลับยาวแต่ดันตื่น ในตลาดกำลังฉลองกันหลายร้านเลย ชวนผมดื่มเบียร์ด้วย 555 ขอบายล่ะครับ. ของมึนเมาเราไม่แตะ

สวัสดีปีใหม่ ชาว mathcenter ทุกคนเช่นกันครับ. :D ยู้ฮู ..... หวังว่าปีนี้ผมและสมาชิกทุกท่านจะขยันและมีความสนุกกับสิ่งที่ตัวเองทำกว่าปีที่แล้ว ...

nooonuii 31 ธันวาคม 2004 23:39

โอ๊ะโอ ผ่านไปแป๊บเดียว คุณ aaaa คิดออกไปสองข้อแล้วครับ เก่งจริงๆ
งั้นผมคงต้องคิดโจทย์เพิ่มแล้วสิครับเนี่ย สวัสดีปีใหม่อีกครั้งครับ ปีเก่ากำลังจะผ่านไปในไม่กี่นาทีข้างหน้านี้แล้วครับ :)

R-Tummykung de Lamar 01 มกราคม 2005 00:14

สวัสดีปีใหม่คร้าบบบบ :D :D :p :p :) :) ;) ;)

<aaaa> 01 มกราคม 2005 01:13

เฉลยข้อ 6
กรณีทั่วไป (1+2+...+n)(1+1/2+...1/n)>(n+1)^2 เมื่อ n>6
อสมการสมมูลกับ 1+1/2+...+1/n>2(1+1/n) เนื่องจากเมื่อ n>6, 1+1/2+...+1/n>2.5 แต่ 2(1+1/n)<2.5

gon 01 มกราคม 2005 01:34

ขอสักข้อล่ะกันครับ.

ข้อ 6 จะแสดงว่า
\((1+2+3+...+n)(1+\frac{1}{2}+\frac{1}{3}+...+\frac{1}{n}) > (n+1)^2 , ทุก\, n \geq 6\)

เนื่องจาก \(L.H.S. = (1+\frac{1}{2}+\frac{1}{3}+\frac{1}{4})(\frac{n^2+n}{2}) + (\frac{1}{5}+\frac{1}{6}+...+\frac{1}{n})(\frac{n^2+n}{2})\)
\( = \frac{25}{12}(\frac{n^2+n}{2}) + (\frac{1}{5}+\frac{1}{6}+...+\frac{1}{n})(\frac{n^2+n}{2})\)
\( > 2(\frac{n^2+n}{2}) + (\frac{1}{5}+\frac{1}{6}+...+\frac{1}{n})(\frac{n^2+n}{2})\)
\( > n^2 + n + (\frac{1}{n}+\frac{1}{n}+...+\frac{1}{n})(\frac{n^2+n}{2}) จำนวน \, n - 4 พจน์\)
\( = n^2 + n + \frac{(n-4)(n+1)}{2}\)
\( = \frac{3n^2-n-4}{2} \geq n^2 + 2n + 1\)
\( \Leftrightarrow 3n^2 - n - 4 \geq 2n^2 + 4n + 2\)
\( \Leftrightarrow n^2 - 5n - 6 \geq 0\)
\( \Leftrightarrow (n-6)(n+1) \geq 0\)
\( \Leftrightarrow n \geq 6\)

gon 01 มกราคม 2005 01:38

อ้าว. คุณ aaaa : จัดการ solve ไปแล้วนี่ เหอ ๆ :p

gon 01 มกราคม 2005 02:55

โอ้. เหนื่อยจริง ๆ สำหรับโจทย์ข้อ 2 ถ้านับเลขไม่ผิดจะมี 72 ชุด คือ

ในทุกกรณี O = 9, R = 0

กรณีที่ 1 : N = 3
กรณีที่ 1.1 : (U, V) = (5, 8) หรือ (8, 5) ในแต่ละแบบ
จะมี (I, E) ได้ 12 ชุด คือ (I, E) {2, 4, 6, 7}
กรณีที่ 1.2 : (U, V) = (6, 7) หรือ (7, 6) ในแต่ละแบบ
จะมี (I, E) ได้ 12 ชุด คือ (I, E) {2, 4, 5, 8}

กรณีที่ 2 : N = 5
จะได้ว่า (U, V) = (7, 8) หรือ (8, 7) ในแต่ละแบบ
จะมี (I, E) ได้ 12 ชุด คือ (I, E) {1, 3, 4, 6}

\ จะมีชุดคำตอบทั้งหมด 24 + 24 + 24 = 72 ชุด

หวังว่าคงไม่ลืมอะไรตกหล่น :)

warut 01 มกราคม 2005 02:56

สวัสดีปีใหม่ด้วยคนคร้าบ ขอให้อะไรที่ดีๆบังเกิดขึ้นกับทุกๆคนนะครับ :)

ไหนๆก็มาที่หัวข้อนี้แล้วก็คงต้องทำโจทย์ของคุณ nooonuii สักข้อ เอาเป็นข้อ 5 ละกัน

ถ้า a S เราจะได้ว่า a, a2, a3, ... จะต้องเป็นสมาชิกของ S ด้วย
แต่ S เป็นเซ็ตจำกัด แสดงว่าค่าของ a, a2, a3, ... จะต้องเป็น periodic ในที่สุด
ดังนั้นค่าของ a ที่เป็นไปได้คือ -1, 0, 1
สรุปว่าเซ็ตคำตอบของโจทย์ข้อนี้คือ {{0}, {1}, {0, 1}, {-1, 1}, {-1, 0, 1}}
ถ้านับเซ็ตว่างด้วยก็เพิ่ม เข้าไปอีกอันนะครับ

nooonuii 01 มกราคม 2005 05:50

หายไปทานข้าวเที่ยงแป๊บเดียว โจทย์ผมขายเกือบหมดแล้วอ่ะ เหลือข้อสามข้อเดียว ซึ่งก็ไม่ยากครับ เดี๋ยวค่อยมาต่อข้อต่อไปให้นะครับ

เอ...ข้อหก ของพี่กร มันแปลกๆนะครับ ดูเหมือนจะมี error นิดหน่อยครับ

Tony 01 มกราคม 2005 13:18

ข้อ 3 [-1,2) รึปล่าวครับ

HAPPY NEW YEAR

nooonuii 01 มกราคม 2005 22:50

คำตอบข้อ 3 ของน้อง Tony ยังไม่ถูกนะครับ เพราะ -1 ไม่ใช่คำตอบแน่นอนครับ แต่ใกล้แล้วครับ

nooonuii 01 มกราคม 2005 23:02

7. จงพิสูจน์โดยไม่ใช้ Fermat's Last Theorem ว่า ไม่มีจำนวนเฉพาะ p,q,r ซึ่งสอดคล้องสมการ pn + qn = rn ทุกค่า n2

8. ให้ x,y,z เป็นจำนวนจริงบวก ซึ่ง xyz = 1 จงพิสูจน์ว่า
\[ (\frac{x^{2548}}{1+x+xy} + \frac{y^{2548}}{1+y+yz} + \frac{z^{2548}}{1+z+zx}) (\frac{y^{2548}}{1+x+xy} + \frac{z^{2548}}{1+y+yz} + \frac{x^{2548}}{1+z+zx}) (\frac{z^{2548}}{1+x+xy} + \frac{x^{2548}}{1+y+yz} + \frac{y^{2548}}{1+z+zx}) \geq 1 \]

gon 02 มกราคม 2005 00:29

ข้อ 2. นี่หาที่ผิดไม่เจอจริง ๆ nooonuii ช่วยชี้แนะด้วยครับ.

R-Tummykung de Lamar 02 มกราคม 2005 02:31

ข้อ 3 ตอบว่า [0,2) ถูกไหมครับ

R-Tummykung de Lamar 02 มกราคม 2005 10:11

ปัญหาหนอนแทะครับ
F I V E O U R N
1 2 5 4 9 8 0 3
1 2 5 6 9 8 0 3
1 2 5 7 9 8 0 3
1 2 6 4 9 7 0 3
1 2 6 5 9 7 0 3
1 2 6 8 9 7 0 3
1 2 7 4 9 6 0 3
1 2 7 5 9 6 0 3
1 2 7 8 9 6 0 3
1 2 8 4 9 5 0 3
1 2 8 6 9 5 0 3
1 2 8 7 9 5 0 3
1 4 5 2 9 8 0 3
1 4 5 6 9 8 0 3
1 4 5 7 9 8 0 3
1 4 6 2 9 7 0 3
1 4 6 5 9 7 0 3
1 4 6 8 9 7 0 3
1 4 7 2 9 6 0 3
1 4 7 5 9 6 0 3
1 4 7 8 9 6 0 3
1 4 8 2 9 5 0 3
1 4 8 6 9 5 0 3
1 4 8 7 9 5 0 3
1 5 6 2 9 7 0 3
1 5 6 4 9 7 0 3
1 5 6 8 9 7 0 3
1 5 7 2 9 6 0 3
1 5 7 4 9 6 0 3
1 5 7 8 9 6 0 6
1 6 5 2 9 8 0 3
1 6 5 4 9 8 0 3
1 6 5 7 9 8 0 3
1 6 8 2 9 5 0 3
1 6 8 4 9 5 0 3
1 6 8 7 9 5 0 3
1 7 5 2 9 8 0 3
1 7 5 4 9 8 0 3
1 7 5 6 9 8 0 3
1 7 8 2 9 5 0 3
1 7 8 4 9 5 0 3
1 7 8 6 9 5 0 3
1 8 6 2 9 7 0 3
1 8 6 4 9 7 0 3
1 8 6 5 9 7 0 3
1 8 7 2 9 6 0 3
1 8 7 4 9 6 0 3
1 8 7 5 9 6 0 3
2 1 7 3 9 8 0 5
2 1 7 4 9 8 0 5
2 1 7 6 9 8 0 5
2 1 8 3 9 7 0 5
2 1 8 4 9 7 0 5
2 1 8 6 9 7 0 5
2 3 7 1 9 8 0 5
2 3 7 4 9 8 0 5
2 3 7 6 9 8 0 5
2 3 8 1 9 7 0 5
2 3 8 4 9 7 0 5
2 3 8 6 9 7 0 5
2 4 7 1 9 8 0 5
2 4 7 3 9 8 0 5
2 4 7 6 9 8 0 5
2 4 8 1 9 7 0 5
2 4 8 3 9 7 0 5
2 4 8 6 9 7 0 5
2 6 7 1 9 8 0 5
2 6 7 3 9 8 0 5
2 6 7 4 9 8 0 5
2 6 8 1 9 7 0 5
2 6 8 3 9 7 0 5
2 6 8 4 9 7 0 5


แฮ่กๆๆ เหนื่อยเลยครับ :D
ใช้ computer serch(เลียนแบบคุณ warut) ครับ :D

aaaa 02 มกราคม 2005 10:12

เฉลยข้อ 8
สังเกตุว่า
\[1=\frac{1}{1+x+xy}+\frac{1}{1+y+yz}+\frac{1}{1+z+zx}\]
ใช้ Holder's inequality ได้คำตอบตามต้องการ
P.S. ข้อนี้ไม่จำเป็นต้องยกกำลัง 2548 จะเป็นกำลัง n ใดๅก็ได้

nooonuii 02 มกราคม 2005 12:09

ขอโทษครับพี่กร ทุกอย่างสมบูรณ์แบบแล้วครับ เป็นความผิดของผมเองครับ เบลอไปหน่อย :D

nooonuii 02 มกราคม 2005 12:48

9. จงหาพื้นที่ของรูปสี่เหลี่ยมนูน (convex quadrilateral) ซึ่งมีความยาวด้านเป็น 1,3,4,5 หน่วย และมีเส้นทแยงมุมเส้นหนึ่งยาว 5 หน่วย

10. จงพิสูจน์ว่า ทุกปีจะต้องมีวันศุกร์ที่ 13 อย่างน้อยหนึ่งวัน

11. จงหาจำนวนจุดที่น้อยที่สุดซึ่งเมื่อบรรจุในวงกลมหนึ่งหน่วย(รวมขอบวงกลม) แล้วจะต้องมีอย่างน้อยสองจุดที่ห่างกันไม่เกินหนึ่งหน่วย

12. จงพิสูจน์ว่าสมการ x!y! = z! มีคำตอบที่เป็นจำนวนเต็มบวกทั้งหมดเป็นจำนวนอนันต์ เมื่อ x>5

nooonuii 02 มกราคม 2005 13:07

13. ให้ a,b,c เป็นจำนวนจริงบวก จงพิสูจน์ว่า

1) (IMO'1995) ถ้า abc = 1 แล้ว 1 / a3(b + c) + 1/b3(c + a) + 1/c3(a + b) 3/2

2) (nooonuii) ถ้า abc = 1 แล้ว 1 / a2(b + c) + 1/b2(c + a) + 1/c2(a + b) 3/2

3) (nooonuii) ถ้า a + b + c = 3 แล้ว 1 / a(b + c) + 1/b(c + a) + 1/c(a + b) 3/2

aaaa 02 มกราคม 2005 13:09

ข้อ 12 นี่ผิดพลาดรึเปล่าครับ ถ้า x=0 มันเห็นชัดว่า y=z เป็นจำนวนเต็มไม่ติดลบใดๆก็ได้

aaaa 02 มกราคม 2005 13:20

เฉลยข้อ 11
ตอบ 6 จุด โดยการแบ่งวงกลมหนึ่งหน่วย ออกเป็น 6 ส่วนเท่าๆกันด้วยเส้นตรงหกเส้นที่ผ่านจุดศก
ห่างกันทีละ 60 องศา และประยุกต์ pigeonholes principle

aaaa 02 มกราคม 2005 13:25

เฉลย ข้อ 9
ตอบ \( 6+\frac{3\sqrt{11}}{4}\)

aaaa 02 มกราคม 2005 16:03

เฉลยข้อ 13
(i) โดยอสมการ Cauchy-Schwarz ได้ว่า
\[\left(\frac{1}{a^3(b+c)}+\frac{1}{b^3(c+a)}+\frac{1}{c^3(a+b)}\right)\left(a(b+c)+b(c+a)+c(a+b)\right)
\geq\left(\frac{1}{a}+\frac{1}{b}+\frac{1}{c}\right)^2=(ab+bc+ca)^2\]
ดังนั้น \(\text{LHS}\geq\frac{ab+bc+ca}{2}\) ใช้อสมการ AM-GM ได้ผลที่ต้องการ

(ii) พิจารณาเทอม
\[\frac{1}{a^2(b+c)}=\frac{abc}{a^2(b+c)}=\frac{bc}{ca+ab}\]
ทำนองเดียวกันได้ว่า
\[\frac{1}{b^2(c+a)}=\frac{ca}{ab+bc},\quad\frac{1}{c^2(a+b)}=\frac{ab}{bc+ca}\]
ให้ \(x=bc,y=ca,z=ab\) ได้อสมการโจทย์สมมูลกับ \(xyz=1\) และ
\[\frac{x}{y+z}+\frac{y}{z+x}+\frac{z}{x+y}\geq\frac{3}{2}
\]
ซึ่งพิสูจน์ได้ทำนองเดียวกับข้อ 13(i)

(iii) โดยอสมการ Holder ได้ว่า
\[\left(\frac{1}{a(b+c)}+\frac{1}{b(c+a)}+\frac{1}{c(a+b)}\right)\left(a+b+c\right)\left((b+c)+(c+a)+(a+b)\right)\geq
(1+1+1)^3
\]
ดังนั้น
\[\frac{1}{a(b+c)}+\frac{1}{b(c+a)}+\frac{1}{c(a+b)}\geq\frac{3}{2}
\]

nooonuii 02 มกราคม 2005 22:40

แก้ข้อ 12 ให้แล้วครับคุณ aaaa
ส่วนข้อ 11 ยังทำให้น้อยกว่านั้นได้ (มั้ง) ครับ :D ผมก็ไม่แน่ใจเหมือนกันว่าคำตอบผมถูกรึเปล่า แต่ผมลดลงมาได้ถึง 5 ครับ

ข้อ 9 คำตอบน่าจะเป็น 6+3/2 11 นะครับ
ข้อ 3 ของน้อง R-Tummykung de Lamar ถูกแล้วครับ

aaaa 03 มกราคม 2005 00:04

ข้อ 9 ผม check แล้วครับ พื้นที่สามเหลี่ยมส่วนที่มีด้านยาว 1,5,5 ส่วนสูงเท่ากับ \(\sqrt{25-\frac{1}{4}}=\frac{3\sqrt{11}}{2}\)
ดังนั้นพื้นที่เท่ากับ \(\frac{1}{2}\times1\times\frac{3\sqrt{11}}{2}=\frac{3\sqrt{11}}{4}\) ของผมถูกแล้วครับ

ข้อ 11 ถ้า 5 จุดผมทำให้แต่ละจุดห่างกันมากกว่า 1 หน่วยได้ครับ ลองนึกถึงรูปหกเหลี่ยมยาวด้านละ 1 หน่วย
ที่แนบในวงกลมหนึ่งหน่วย เลือกจุด 5 จุดให้ห่างกันมากกว่า 1 หน่วยได้ครับ

ข้อ 12 ถ้า x=1 ยังคงเลือกให้ \(y=z>0\) ใดๆก็ได้

aaaa 03 มกราคม 2005 00:12

ผมมีโจทย์มาถามครับ (Putnam Exam 1966)
ข้อ 1 จงหา
\[
\lim_{n\to\infty}\sqrt{1+2\sqrt{1+3\sqrt{1+\sqrt{\cdots+(n-1)\sqrt{1+n}}}}}
\]

warut 03 มกราคม 2005 00:32

คุณ TOP เคยแสดงวิธีทำโจทย์แบบเดียวกันนี้อย่างเหนือชั้นไว้เมื่อนานมาแล้วครับ

aaaa 03 มกราคม 2005 01:33

ครับสุดยอดจริงๆสำหรับแนวคิด เอาเป็นว่าผมขอเสนอวิธีของผมด้วยละกัน

ให้ \(a_n\) แทนเทอมที่ต้องการหาลิมิต จะเห็นได้ว่า \(a_n\) เป็นลำดับเพิ่ม และยิ่งกว่านั้น \(a_n<3\) เนื่องจากหากแทนพจน์ \(\sqrt{1+n}\) ด้วย \(1+n\) ในสูตรของ \(a_n\) ค่าที่ได้(ซึ่งเพิ่มขึ้น) จะเท่ากับ 3 พอดี ดังนั้นลำดับ \(a_n\) ลู่เข้า และ \(\lim_{n\to\infty}a_n\leq3\)

พิจารณาลำดับ
\[
b_N=\sqrt{1+N\sqrt{1+(N+1)\sqrt{1+\cdots}}}
\]
ซึ่งพิสูจน์ได้ทำนองเดียวกับข้างบนว่า \(b_N\) ลู่เข้า จะเห็นว่าถ้าตัด 1 ออกจากสูตรของ \(b_N\) ค่าที่ได้จะน้อยลง ดังนั้น
\[
b_N\geq\sqrt{N\sqrt{(N+1)\sqrt{(N+2)\sqrt{\cdots}}}}\geq\sqrt{N(N+1)}
\]

ดังนั้นเราได้ว่า
\[
\lim_{n\to\infty}a_n=\sqrt{1+2\sqrt{1+3\sqrt{1+\cdots\sqrt{1+(N-1)b_N}}}}
\]

แต่เนื่องจาก \(\sqrt{N(N+1)}=O(N+1)\) เมื่อ \(N\) มากๆ และ หากแทนพจน์ \(b_N\) ด้วย \(N+1\) ค่าที่ได้เท่ากับ 3 พอดี ดังนั้นเราสรุปได้ว่า \(\lim_{n\to\infty}a_n\geq3\)

nooonuii 03 มกราคม 2005 10:33

อืมผมคิดผิดจริงๆด้วยนั่นแหละครับคุณ aaaa
ส่วนข้อ 12 นี่ยังง่ายจริงๆด้วยครับ งั้นผมเพิ่มเงื่อนไขเป็น x>5 แล้วกันนะครับเพื่อขจัด trivial solution ไป จริงๆข้อนี้อยากให้รู้จักเอกลักษณ์ที่น่าสนใจเกี่ยวกับ factorial น่ะครับ สวยดี

aaaa 03 มกราคม 2005 11:09

เฉลยข้อ 12
ให้ \(y=x!-1,\,z=x!\)

aaaa 03 มกราคม 2005 12:15

กรณีทั่วไปของข้อ 13
ให้ \(n\geq3\) เป็นจำนวนจริงใดๆ จงพิสูจน์ว่า ถ้า \(a,b,c>0\) และ \(abc=1\) แล้ว
\[
\frac{1}{a^n(b+c)}+\frac{1}{b^n(c+a)}+\frac{1}{c^n(a+b)}\geq\frac{3}{2}
\]

aaaa 03 มกราคม 2005 15:16

อืมผมเพิ่งจะพบว่าโจทย์ข้อนี้มีคนทำไปแล้วในเวปอื่น
ขออนุญาตินำเสนอวิธีที่เขานำเสนอครับ

ให้ \(x=1/a,\,y=1/b,\,z=1/c\) จะได้อสมการสมมูลกับ
\[
\frac{x^{n-1}}{y+z}+\frac{y^{n-1}}{z+x}+\frac{z^{n-1}}{x+y}\geq\frac{3}{2}
\]
เนื่องจากอสมการสมมาตรเมื่อสลับลำดับ (cyclic) เราสามารถสมมติว่า \(x\leq y\leq z\) ดังนั้นโดยอสมการ Chebyshev จะได้ว่า
\[
\frac{x^{n-1}}{y+z}+\frac{y^{n-1}}{z+x}+\frac{z^{n-1}}{x+y}\geq\frac{1}{3}\left(x^{n-2}+y^{n-2}+z^{n-2}\right)\left(\frac{x}{y+z}
+\frac{y}{z+x}+\frac{z}{z+y}\right)
\]
เทอมแรกใช้อสมการ Power-Mean และอสมการ AM-GM ได้ว่า
\[
\left(x^{n-2}+y^{n-2}+z^{n-2}\right)\geq(x+y+z)^{n-2}/3^{n-3}\geq\frac{3^{n-2}}{3^{n-3}}=3
\]
เทอมที่เหลือใช้อสมการ Cauchy

gon 03 มกราคม 2005 16:48

เยี่ยมครับ. ได้คุณ aaaa มาเล่นอีกคนครึกครื้นขึ้นเยอะเลย คิดเร็วมาก ๆ เลยครับ. อย่างกับซุบเปอร์ไซยา เรื่องอสมการเราเคยเล่นกันมาบ้างแล้วครับ. ในกระทู้นี้ โจทย์อสมการ

นี่โจทย์เก่าครับ. ขุดขึ้นมาเขียนแบบ Latex ใหม่
สำหรับทุกจำนวนจริงบวก \(a,b,c\) โดยที่ \(abc = 1\) จงพิสูจน์ว่า
\[\frac{a^3b^3}{a^7+b^7+a^3b^3} + \frac{b^3c^3}{b^7+c^7+b^3c^3} + \frac{c^3a^3}{c^7+a^7+c^3a^3} \leq 1\]

aaaa 03 มกราคม 2005 18:15

ขอบคุณครับคุณ gon ตั้งแต่รู้จักเวปนี้สนุกมากครับ
มีประโยชน์มากเลยครับอยากให้เวปนี้อยู่ตลอดไปครับ
จะได้เป็นแหล่งแลกเปลี่ยนความรู้ด้าน math ของคนไทย

gon 03 มกราคม 2005 20:34

ข้อกรณีทั่วไปของ ข้อ 13. ผมเสนอแนวคิดซึ่งสลับนิดหน่อยกับของเก่าดังนี้ครับ.
หลังจากจัดรูป \(LHS.\) ได้เป็น
\[\frac{x^{n-1}}{y+z} + \frac{y^{n-1}}{z+x} + \frac{z^{n-1}}{x+y}\]
จากนั้นทำแบบนี้ต่อคือ โดยโคชี \(
(LHS.)[ (y+z) + (z+x) + (x+y) ] \geq (x^{\frac{n-1}{2}} + y^{\frac{n-1}{2}} + + z^{\frac{n-1}{2}})^2\)
\
\[LHS. \geq \frac{(x^{\frac{n-1}{2}} + y^{\frac{n-1}{2}} + z^{\frac{n-1}{2}})^2}{2(x+y+z)} \geq \frac{(x+y+z)^2}{2(x+y+z)} = \frac{x+y+z}{2} \geq \frac{3}{2}\sqrt[3]{xyz} = \frac{3}{2}\]

เพราะเมื่อ \(x, y, z > 0, xyz = 1, m \geq 1\)
\[x^m + y^m + z^m \geq x + y + z\]
เพราะ โดย Chebyshev
\(x^m + y^m + z^m \geq \frac{(x^{m-1}+y^{m-1}+z^{m-1})(x+y+z)}{3} \geq \sqrt[3]{(xyz)^{m-1}}(x+y+z) = x + y + z\)

aaaa 03 มกราคม 2005 21:04

เยี่ยมจริงๆครับคุณ gon โดยเฉพาะอสมการรองสุดท้าย


เวลาที่แสดงทั้งหมด เป็นเวลาที่ประเทศไทย (GMT +7) ขณะนี้เป็นเวลา 11:07

Powered by vBulletin® Copyright ©2000 - 2024, Jelsoft Enterprises Ltd.
Modified by Jetsada Karnpracha